Evaluating big-O vs big-Omega for two functions












0












$begingroup$


We were tasked with comparing the complexities of two functions: $n$ and $n^{0.99} (log(n))^2$.



As I understand it, the general construct of these equations is $lim_{x rightarrow infty} frac{f(n)}{g(n)}$. If the limit evaluates to 0, we have $f in O(g)$. If the limit evaluates to infinity, $f in Omega(g)$. If it evaluates to a constant, then $f in Theta(g)$.



If we plot the quotient of these two functions, then we clearly see that the result approaches 0:
Wolfram Alpha Plot



However, if we plug the limit into Wolfram Alpha, we get
$lim_{x rightarrow infty} frac{x}{x^{0.99} (log(x))^2} = infty$



These two results seem to be contradictory. Interestingly enough, if we ask for $lim_{x rightarrow infty} frac{d}{dx}frac{x}{x^{0.99} (log(x))^2}$, Wolfram Alpha gives 0. I don't think it's possible for a limit to be infinity and the derivative's limit to be 0...



I must be missing something, but I can't figure out what. Any ideas?



Edit: Got it, graphs are useless in this scenario. However, suppose I had to do this calculation manually. How would I go about evaluating the limit?










share|cite|improve this question











$endgroup$












  • $begingroup$
    Looking at a graph proves nothing, as you can see.
    $endgroup$
    – Matt Samuel
    Jan 21 at 23:56
















0












$begingroup$


We were tasked with comparing the complexities of two functions: $n$ and $n^{0.99} (log(n))^2$.



As I understand it, the general construct of these equations is $lim_{x rightarrow infty} frac{f(n)}{g(n)}$. If the limit evaluates to 0, we have $f in O(g)$. If the limit evaluates to infinity, $f in Omega(g)$. If it evaluates to a constant, then $f in Theta(g)$.



If we plot the quotient of these two functions, then we clearly see that the result approaches 0:
Wolfram Alpha Plot



However, if we plug the limit into Wolfram Alpha, we get
$lim_{x rightarrow infty} frac{x}{x^{0.99} (log(x))^2} = infty$



These two results seem to be contradictory. Interestingly enough, if we ask for $lim_{x rightarrow infty} frac{d}{dx}frac{x}{x^{0.99} (log(x))^2}$, Wolfram Alpha gives 0. I don't think it's possible for a limit to be infinity and the derivative's limit to be 0...



I must be missing something, but I can't figure out what. Any ideas?



Edit: Got it, graphs are useless in this scenario. However, suppose I had to do this calculation manually. How would I go about evaluating the limit?










share|cite|improve this question











$endgroup$












  • $begingroup$
    Looking at a graph proves nothing, as you can see.
    $endgroup$
    – Matt Samuel
    Jan 21 at 23:56














0












0








0





$begingroup$


We were tasked with comparing the complexities of two functions: $n$ and $n^{0.99} (log(n))^2$.



As I understand it, the general construct of these equations is $lim_{x rightarrow infty} frac{f(n)}{g(n)}$. If the limit evaluates to 0, we have $f in O(g)$. If the limit evaluates to infinity, $f in Omega(g)$. If it evaluates to a constant, then $f in Theta(g)$.



If we plot the quotient of these two functions, then we clearly see that the result approaches 0:
Wolfram Alpha Plot



However, if we plug the limit into Wolfram Alpha, we get
$lim_{x rightarrow infty} frac{x}{x^{0.99} (log(x))^2} = infty$



These two results seem to be contradictory. Interestingly enough, if we ask for $lim_{x rightarrow infty} frac{d}{dx}frac{x}{x^{0.99} (log(x))^2}$, Wolfram Alpha gives 0. I don't think it's possible for a limit to be infinity and the derivative's limit to be 0...



I must be missing something, but I can't figure out what. Any ideas?



Edit: Got it, graphs are useless in this scenario. However, suppose I had to do this calculation manually. How would I go about evaluating the limit?










share|cite|improve this question











$endgroup$




We were tasked with comparing the complexities of two functions: $n$ and $n^{0.99} (log(n))^2$.



As I understand it, the general construct of these equations is $lim_{x rightarrow infty} frac{f(n)}{g(n)}$. If the limit evaluates to 0, we have $f in O(g)$. If the limit evaluates to infinity, $f in Omega(g)$. If it evaluates to a constant, then $f in Theta(g)$.



If we plot the quotient of these two functions, then we clearly see that the result approaches 0:
Wolfram Alpha Plot



However, if we plug the limit into Wolfram Alpha, we get
$lim_{x rightarrow infty} frac{x}{x^{0.99} (log(x))^2} = infty$



These two results seem to be contradictory. Interestingly enough, if we ask for $lim_{x rightarrow infty} frac{d}{dx}frac{x}{x^{0.99} (log(x))^2}$, Wolfram Alpha gives 0. I don't think it's possible for a limit to be infinity and the derivative's limit to be 0...



I must be missing something, but I can't figure out what. Any ideas?



Edit: Got it, graphs are useless in this scenario. However, suppose I had to do this calculation manually. How would I go about evaluating the limit?







limits algorithms






share|cite|improve this question















share|cite|improve this question













share|cite|improve this question




share|cite|improve this question








edited Jan 22 at 0:04







elder4222

















asked Jan 21 at 23:50









elder4222elder4222

134




134












  • $begingroup$
    Looking at a graph proves nothing, as you can see.
    $endgroup$
    – Matt Samuel
    Jan 21 at 23:56


















  • $begingroup$
    Looking at a graph proves nothing, as you can see.
    $endgroup$
    – Matt Samuel
    Jan 21 at 23:56
















$begingroup$
Looking at a graph proves nothing, as you can see.
$endgroup$
– Matt Samuel
Jan 21 at 23:56




$begingroup$
Looking at a graph proves nothing, as you can see.
$endgroup$
– Matt Samuel
Jan 21 at 23:56










3 Answers
3






active

oldest

votes


















0












$begingroup$

Two further points.



First, your definition of big-O notation is not quite right.




  • If the limit $lim_{n to infty} frac{f(n)}{g(n)}$ evaluates to $0$, then $f in o(g)$; we have $f in O(g)$ if the limit is either $0$ or a constant. In other words, $O(g) = o(g) cup Theta(g)$. Similarly, we write $f in omega(g)$ when the limit is $infty$ and $Omega(g) = Theta(g) cup omega(g)$. A function in $Theta(g)$ is in both $O(g)$ and $Omega(g)$, but isn't in $o(g)$ or $omega(g)$.


  • Technically, we can also have $f in Theta(g)$ if the limit $frac{f(n)}{g(n)}$ as $n to infty$ doesn't exist because of oscillation, but the ratio is still bounded for all sufficiently large $n$. For example, we want $1 + frac12 sin n$ to be $Theta(1)$, though the limit doesn't exist here. But this doesn't usually come up much.



Second, to answer your question of how we'd figure this out.



First, we should simplify as much as possible. Comparing $n$ to $n^{0.99} (log n)^2$ can be reduced to comparing $n^{0.01}$ to $(log n)^2$, which can in turn be reduced to comparing $n^{0.005}$ to $log n$.



In practice, you just learn the asymptotic comparisons between common functions. One of them is that $log n in o(n^alpha)$ for any $alpha > 0$, which answers this question. But if we wanted to understand the comparison above better, we could also substitute $n = k^{200}$, which gives us the comparison $(k^{200})^{0.005} = k$ vs. $log (k^{200}) = 200log k$. This comparison goes the same way as $k$ vs. $log k$, which is hopefully more clear-cut.






share|cite|improve this answer









$endgroup$





















    1












    $begingroup$

    The plot is misleading: $x$ must become quite large before $x^{0.01}$ dominates over $log(x)^2$, but eventually it does. The limit query is indeed correct.



    As for the derivative, no, that's not the case, consider for a simpler example $x^alpha$ for any $0<alpha<1$.






    share|cite|improve this answer









    $endgroup$





















      0












      $begingroup$

      $$frac{x^{0.01}}{log^2x}$$ evaluated at $x=e^{10000}$ gives about



      $$2.7cdot10^{35}.$$



      Not so close to $0$...






      share|cite|improve this answer









      $endgroup$













        Your Answer





        StackExchange.ifUsing("editor", function () {
        return StackExchange.using("mathjaxEditing", function () {
        StackExchange.MarkdownEditor.creationCallbacks.add(function (editor, postfix) {
        StackExchange.mathjaxEditing.prepareWmdForMathJax(editor, postfix, [["$", "$"], ["\\(","\\)"]]);
        });
        });
        }, "mathjax-editing");

        StackExchange.ready(function() {
        var channelOptions = {
        tags: "".split(" "),
        id: "69"
        };
        initTagRenderer("".split(" "), "".split(" "), channelOptions);

        StackExchange.using("externalEditor", function() {
        // Have to fire editor after snippets, if snippets enabled
        if (StackExchange.settings.snippets.snippetsEnabled) {
        StackExchange.using("snippets", function() {
        createEditor();
        });
        }
        else {
        createEditor();
        }
        });

        function createEditor() {
        StackExchange.prepareEditor({
        heartbeatType: 'answer',
        autoActivateHeartbeat: false,
        convertImagesToLinks: true,
        noModals: true,
        showLowRepImageUploadWarning: true,
        reputationToPostImages: 10,
        bindNavPrevention: true,
        postfix: "",
        imageUploader: {
        brandingHtml: "Powered by u003ca class="icon-imgur-white" href="https://imgur.com/"u003eu003c/au003e",
        contentPolicyHtml: "User contributions licensed under u003ca href="https://creativecommons.org/licenses/by-sa/3.0/"u003ecc by-sa 3.0 with attribution requiredu003c/au003e u003ca href="https://stackoverflow.com/legal/content-policy"u003e(content policy)u003c/au003e",
        allowUrls: true
        },
        noCode: true, onDemand: true,
        discardSelector: ".discard-answer"
        ,immediatelyShowMarkdownHelp:true
        });


        }
        });














        draft saved

        draft discarded


















        StackExchange.ready(
        function () {
        StackExchange.openid.initPostLogin('.new-post-login', 'https%3a%2f%2fmath.stackexchange.com%2fquestions%2f3082575%2fevaluating-big-o-vs-big-omega-for-two-functions%23new-answer', 'question_page');
        }
        );

        Post as a guest















        Required, but never shown

























        3 Answers
        3






        active

        oldest

        votes








        3 Answers
        3






        active

        oldest

        votes









        active

        oldest

        votes






        active

        oldest

        votes









        0












        $begingroup$

        Two further points.



        First, your definition of big-O notation is not quite right.




        • If the limit $lim_{n to infty} frac{f(n)}{g(n)}$ evaluates to $0$, then $f in o(g)$; we have $f in O(g)$ if the limit is either $0$ or a constant. In other words, $O(g) = o(g) cup Theta(g)$. Similarly, we write $f in omega(g)$ when the limit is $infty$ and $Omega(g) = Theta(g) cup omega(g)$. A function in $Theta(g)$ is in both $O(g)$ and $Omega(g)$, but isn't in $o(g)$ or $omega(g)$.


        • Technically, we can also have $f in Theta(g)$ if the limit $frac{f(n)}{g(n)}$ as $n to infty$ doesn't exist because of oscillation, but the ratio is still bounded for all sufficiently large $n$. For example, we want $1 + frac12 sin n$ to be $Theta(1)$, though the limit doesn't exist here. But this doesn't usually come up much.



        Second, to answer your question of how we'd figure this out.



        First, we should simplify as much as possible. Comparing $n$ to $n^{0.99} (log n)^2$ can be reduced to comparing $n^{0.01}$ to $(log n)^2$, which can in turn be reduced to comparing $n^{0.005}$ to $log n$.



        In practice, you just learn the asymptotic comparisons between common functions. One of them is that $log n in o(n^alpha)$ for any $alpha > 0$, which answers this question. But if we wanted to understand the comparison above better, we could also substitute $n = k^{200}$, which gives us the comparison $(k^{200})^{0.005} = k$ vs. $log (k^{200}) = 200log k$. This comparison goes the same way as $k$ vs. $log k$, which is hopefully more clear-cut.






        share|cite|improve this answer









        $endgroup$


















          0












          $begingroup$

          Two further points.



          First, your definition of big-O notation is not quite right.




          • If the limit $lim_{n to infty} frac{f(n)}{g(n)}$ evaluates to $0$, then $f in o(g)$; we have $f in O(g)$ if the limit is either $0$ or a constant. In other words, $O(g) = o(g) cup Theta(g)$. Similarly, we write $f in omega(g)$ when the limit is $infty$ and $Omega(g) = Theta(g) cup omega(g)$. A function in $Theta(g)$ is in both $O(g)$ and $Omega(g)$, but isn't in $o(g)$ or $omega(g)$.


          • Technically, we can also have $f in Theta(g)$ if the limit $frac{f(n)}{g(n)}$ as $n to infty$ doesn't exist because of oscillation, but the ratio is still bounded for all sufficiently large $n$. For example, we want $1 + frac12 sin n$ to be $Theta(1)$, though the limit doesn't exist here. But this doesn't usually come up much.



          Second, to answer your question of how we'd figure this out.



          First, we should simplify as much as possible. Comparing $n$ to $n^{0.99} (log n)^2$ can be reduced to comparing $n^{0.01}$ to $(log n)^2$, which can in turn be reduced to comparing $n^{0.005}$ to $log n$.



          In practice, you just learn the asymptotic comparisons between common functions. One of them is that $log n in o(n^alpha)$ for any $alpha > 0$, which answers this question. But if we wanted to understand the comparison above better, we could also substitute $n = k^{200}$, which gives us the comparison $(k^{200})^{0.005} = k$ vs. $log (k^{200}) = 200log k$. This comparison goes the same way as $k$ vs. $log k$, which is hopefully more clear-cut.






          share|cite|improve this answer









          $endgroup$
















            0












            0








            0





            $begingroup$

            Two further points.



            First, your definition of big-O notation is not quite right.




            • If the limit $lim_{n to infty} frac{f(n)}{g(n)}$ evaluates to $0$, then $f in o(g)$; we have $f in O(g)$ if the limit is either $0$ or a constant. In other words, $O(g) = o(g) cup Theta(g)$. Similarly, we write $f in omega(g)$ when the limit is $infty$ and $Omega(g) = Theta(g) cup omega(g)$. A function in $Theta(g)$ is in both $O(g)$ and $Omega(g)$, but isn't in $o(g)$ or $omega(g)$.


            • Technically, we can also have $f in Theta(g)$ if the limit $frac{f(n)}{g(n)}$ as $n to infty$ doesn't exist because of oscillation, but the ratio is still bounded for all sufficiently large $n$. For example, we want $1 + frac12 sin n$ to be $Theta(1)$, though the limit doesn't exist here. But this doesn't usually come up much.



            Second, to answer your question of how we'd figure this out.



            First, we should simplify as much as possible. Comparing $n$ to $n^{0.99} (log n)^2$ can be reduced to comparing $n^{0.01}$ to $(log n)^2$, which can in turn be reduced to comparing $n^{0.005}$ to $log n$.



            In practice, you just learn the asymptotic comparisons between common functions. One of them is that $log n in o(n^alpha)$ for any $alpha > 0$, which answers this question. But if we wanted to understand the comparison above better, we could also substitute $n = k^{200}$, which gives us the comparison $(k^{200})^{0.005} = k$ vs. $log (k^{200}) = 200log k$. This comparison goes the same way as $k$ vs. $log k$, which is hopefully more clear-cut.






            share|cite|improve this answer









            $endgroup$



            Two further points.



            First, your definition of big-O notation is not quite right.




            • If the limit $lim_{n to infty} frac{f(n)}{g(n)}$ evaluates to $0$, then $f in o(g)$; we have $f in O(g)$ if the limit is either $0$ or a constant. In other words, $O(g) = o(g) cup Theta(g)$. Similarly, we write $f in omega(g)$ when the limit is $infty$ and $Omega(g) = Theta(g) cup omega(g)$. A function in $Theta(g)$ is in both $O(g)$ and $Omega(g)$, but isn't in $o(g)$ or $omega(g)$.


            • Technically, we can also have $f in Theta(g)$ if the limit $frac{f(n)}{g(n)}$ as $n to infty$ doesn't exist because of oscillation, but the ratio is still bounded for all sufficiently large $n$. For example, we want $1 + frac12 sin n$ to be $Theta(1)$, though the limit doesn't exist here. But this doesn't usually come up much.



            Second, to answer your question of how we'd figure this out.



            First, we should simplify as much as possible. Comparing $n$ to $n^{0.99} (log n)^2$ can be reduced to comparing $n^{0.01}$ to $(log n)^2$, which can in turn be reduced to comparing $n^{0.005}$ to $log n$.



            In practice, you just learn the asymptotic comparisons between common functions. One of them is that $log n in o(n^alpha)$ for any $alpha > 0$, which answers this question. But if we wanted to understand the comparison above better, we could also substitute $n = k^{200}$, which gives us the comparison $(k^{200})^{0.005} = k$ vs. $log (k^{200}) = 200log k$. This comparison goes the same way as $k$ vs. $log k$, which is hopefully more clear-cut.







            share|cite|improve this answer












            share|cite|improve this answer



            share|cite|improve this answer










            answered Jan 22 at 3:44









            Misha LavrovMisha Lavrov

            47k657107




            47k657107























                1












                $begingroup$

                The plot is misleading: $x$ must become quite large before $x^{0.01}$ dominates over $log(x)^2$, but eventually it does. The limit query is indeed correct.



                As for the derivative, no, that's not the case, consider for a simpler example $x^alpha$ for any $0<alpha<1$.






                share|cite|improve this answer









                $endgroup$


















                  1












                  $begingroup$

                  The plot is misleading: $x$ must become quite large before $x^{0.01}$ dominates over $log(x)^2$, but eventually it does. The limit query is indeed correct.



                  As for the derivative, no, that's not the case, consider for a simpler example $x^alpha$ for any $0<alpha<1$.






                  share|cite|improve this answer









                  $endgroup$
















                    1












                    1








                    1





                    $begingroup$

                    The plot is misleading: $x$ must become quite large before $x^{0.01}$ dominates over $log(x)^2$, but eventually it does. The limit query is indeed correct.



                    As for the derivative, no, that's not the case, consider for a simpler example $x^alpha$ for any $0<alpha<1$.






                    share|cite|improve this answer









                    $endgroup$



                    The plot is misleading: $x$ must become quite large before $x^{0.01}$ dominates over $log(x)^2$, but eventually it does. The limit query is indeed correct.



                    As for the derivative, no, that's not the case, consider for a simpler example $x^alpha$ for any $0<alpha<1$.







                    share|cite|improve this answer












                    share|cite|improve this answer



                    share|cite|improve this answer










                    answered Jan 21 at 23:53









                    IanIan

                    68.4k25388




                    68.4k25388























                        0












                        $begingroup$

                        $$frac{x^{0.01}}{log^2x}$$ evaluated at $x=e^{10000}$ gives about



                        $$2.7cdot10^{35}.$$



                        Not so close to $0$...






                        share|cite|improve this answer









                        $endgroup$


















                          0












                          $begingroup$

                          $$frac{x^{0.01}}{log^2x}$$ evaluated at $x=e^{10000}$ gives about



                          $$2.7cdot10^{35}.$$



                          Not so close to $0$...






                          share|cite|improve this answer









                          $endgroup$
















                            0












                            0








                            0





                            $begingroup$

                            $$frac{x^{0.01}}{log^2x}$$ evaluated at $x=e^{10000}$ gives about



                            $$2.7cdot10^{35}.$$



                            Not so close to $0$...






                            share|cite|improve this answer









                            $endgroup$



                            $$frac{x^{0.01}}{log^2x}$$ evaluated at $x=e^{10000}$ gives about



                            $$2.7cdot10^{35}.$$



                            Not so close to $0$...







                            share|cite|improve this answer












                            share|cite|improve this answer



                            share|cite|improve this answer










                            answered Jan 22 at 0:03









                            Yves DaoustYves Daoust

                            129k675227




                            129k675227






























                                draft saved

                                draft discarded




















































                                Thanks for contributing an answer to Mathematics Stack Exchange!


                                • Please be sure to answer the question. Provide details and share your research!

                                But avoid



                                • Asking for help, clarification, or responding to other answers.

                                • Making statements based on opinion; back them up with references or personal experience.


                                Use MathJax to format equations. MathJax reference.


                                To learn more, see our tips on writing great answers.




                                draft saved


                                draft discarded














                                StackExchange.ready(
                                function () {
                                StackExchange.openid.initPostLogin('.new-post-login', 'https%3a%2f%2fmath.stackexchange.com%2fquestions%2f3082575%2fevaluating-big-o-vs-big-omega-for-two-functions%23new-answer', 'question_page');
                                }
                                );

                                Post as a guest















                                Required, but never shown





















































                                Required, but never shown














                                Required, but never shown












                                Required, but never shown







                                Required, but never shown

































                                Required, but never shown














                                Required, but never shown












                                Required, but never shown







                                Required, but never shown







                                Popular posts from this blog

                                Mario Kart Wii

                                What does “Dominus providebit” mean?

                                Antonio Litta Visconti Arese